$\def\sen{\text{sen}}$ ======== Integrais impróprias ======== {{ youtube>sNO7LM6H-7o?small}} Dada $f: [a, +\infty[ \to \mathbb R$ integrável em qualquer intervalo $[a, b]$, definimos \[\int_a^{+\infty}f(x)dx = \lim\limits_{t \to +\infty} \int_a^t f(x)dx\] se tal limite existir. No caso em que esse limite é um número real, dizemos que a integral **converge**, caso contrário, dizemos que ela **diverge**. A definição $-\infty$ é análoga. **Exemplo** \[\int_0^{+\infty} e^{-x}dx = \lim\limits_{t \to +\infty} \int_0^t e^{-x}dx = \lim\limits_{t \to +\infty} (-e^{-x})|_0^t = \lim\limits_{t \to +\infty} (1 - e^{-t}) = 1.\] **Exemplo** Se $r \neq 0$, temos que \[\int_0^{+\infty} e^{rx}dx = \lim\limits_{t \to +\infty} \frac{1}{r}(e^{rt} -1).\] Assim, temos que se $r < 0$, tal integral converge para $\frac{-1}{r}$. Se $r > 0$, temos que tal integral vale $+\infty$. O caso $r = 0$ fazemos separadamente: \[\int_0^{+\infty} e^0dx = \lim\limits_{t \to +\infty} t - 0 = +\infty.\] **Exemplo** $\int_0^{+\infty} \sen x dx = \lim\limits_{0 \to +\infty} \cos 0 - \cos t$. Logo, tal integral diverge. **Exemplo** $\int_1^{+\infty} \frac{1}{x} dx = \lim\limits_{t \to +\infty} \ln t - \ln 1 = +\infty$ **Exemplo** Se $p > 0$ e $p \neq 1$, temos \[\int_1^{+\infty} \frac{1}{x^p}dx = \lim\limits_{t \to +\infty} (\frac{1}{1 - p} x^{1 - p})|_1^t = \lim\limits_{t \to +\infty} \frac{1}{1 - p} t^{1 - p} - \frac{1}{1 - p}.\] Assim, se $p > 1$, temos que $1 - p < 0$, logo a integral converge para $\frac{1}{p - 1}$. Se $p < 1$, $1 - p > 0$. Logo a integral vale $+\infty$. Juntando os dois resultados anteriores, temos que **Proposição** Para $p > 0$ \[\int_1^{+ \infty} \frac{1}{x^p} dx= \begin{cases} \frac{1}{p - 1} & \mbox{se } p > 1\\ +\infty & \mbox{se } p \leq 1\\ \end{cases}\] **~~#~~** Esse é um exercício bacana que mistura algumas coisas que fizemos. **~~#.#~~** Calcule $\int_a^b xe^{-x}dx$. **~~#.#~~** Calcule $\int_0^{+\infty} xe^{-x}dx$. **~~#.#~~** Para $n > 1$, calcule $\int_0^{+\infty} x^n e^{-x}dx$. Pelo que desenvolvemos sobre limites, temos que: **Proposição** Sejam $f, g: [a, +\infty[$ integraveis em qualquer intervalo $[a, b]$. Se $0 \leq f(x) \leq g(x)$, então se $\int_a^{+\infty} f(x) = +\infty$, então $\int_a^{+\infty} g(x) = +\infty$. Se $\int_a^{+\infty} g(x) = L \in \mathbb R$, então $\int_a^{+\infty} f(x) \leq L$. Se $f(x) \leq g(x)$ somente a partir de um certo ponto $p$, ainda vale o resultado anterior, basta usar que \[\int_a^{+\infty} f(x) dx = \int_a^p f(x) dx + \int_p^{+ \infty} f(x) dx.\]